Last visit was: 25 Apr 2024, 08:16 It is currently 25 Apr 2024, 08:16

Close
GMAT Club Daily Prep
Thank you for using the timer - this advanced tool can estimate your performance and suggest more practice questions. We have subscribed you to Daily Prep Questions via email.

Customized
for You

we will pick new questions that match your level based on your Timer History

Track
Your Progress

every week, we’ll send you an estimated GMAT score based on your performance

Practice
Pays

we will pick new questions that match your level based on your Timer History
Not interested in getting valuable practice questions and articles delivered to your email? No problem, unsubscribe here.
Close
Request Expert Reply
Confirm Cancel
SORT BY:
Date
Tags:
Show Tags
Hide Tags
Intern
Intern
Joined: 22 Apr 2019
Posts: 26
Own Kudos [?]: 8 [0]
Given Kudos: 13
Send PM
Intern
Intern
Joined: 15 Jul 2019
Posts: 5
Own Kudos [?]: 0 [0]
Given Kudos: 10
Send PM
Intern
Intern
Joined: 22 Apr 2019
Posts: 26
Own Kudos [?]: 8 [0]
Given Kudos: 13
Send PM
avatar
Intern
Intern
Joined: 19 Jun 2019
Posts: 1
Own Kudos [?]: 0 [0]
Given Kudos: 3
Send PM
Re: Inequalities Made Easy! [#permalink]
Hello Bunuel,

How did you factor this?

"Given: −2x3+17x2–30x>0−2x3+17x2–30x>0

x(−2x2+17x–30)>0x(−2x2+17x–30)>0 (taking x common)

x(2x–5)(6–x)>0x(2x–5)(6–x)>0 (factoring the quadratic)"

Thanks
Manager
Manager
Joined: 26 Sep 2013
Posts: 70
Own Kudos [?]: 23 [0]
Given Kudos: 30
GMAT 1: 710 Q47 V40
GMAT 2: 740 Q50 V41
Send PM
Re: Inequalities Made Easy! [#permalink]
Bunuel wrote:
Or Just Use Inequalities!

BY KARISHMA, VERITAS PREP


If you are wondering about the absurd title of this post, just take a look at the above post's title. It will make much more sense thereafter. This post is a continuation of last week’s post where we discussed number plugging. Today, as per students’ request, we will look at the inequalities approach to the same official question. You will need to go through our inequalities post to understand the method we will use here.

Recall that, given \(a < b\), \((x – a)(x – b) < 0\) gives us the range \(a < x < b\) and \((x – a)(x – b) > 0\) gives us the range \(x < a\) or \(x > b\).

Question: If x is positive, which of the following could be the correct ordering of 1/x, 2x and x^2?
(I) x^2 < 2x < 1/x
(II) x^2 < 1/x < 2x
(III) 2x < x^2 < 1/x


(A) none
(B) I only
(C) III only
(D) I and II
(E) I, II and III

Solution: The question has three complex inequalities. We will take each in turn. Note that each inequality consists of two more inequalities. We will split the complex inequality into two simpler inequalities e.g. x^2 < 2x < 1/x gives us x^2 < 2x and 2x < 1/x. Next we will find the range of values of x which satisfy each of these two inequalities and we will see if the two ranges have an overlap i.e. whether there are any values of x which satisfy both these simpler inequalities. If there are, it means there are values of x which satisfy the entire complex inequality too. Things will become clearer once we start working on it so hold on.

Let’s look at each inequality in turn. We start with the first one:

(I) x^2 < 2x < 1/x

We split it into two inequalities:

(i) x^2 < 2x

We can rewrite x^2 < 2x as x^2 – 2x < 0 or x(x – 2) < 0.

We know the range of x for such inequalities can be easily found using the curve on the number line. This will give us 0 < x < 2.

(ii) 2x < 1/x

It can be rewritten as x^2 – 1/2 < 0 (Note that since x must be positive, we can easily multiply both sides of the inequality with x)

This gives us the range -1/?2 < x < 1/?2 (which is 0 < x < 1/?2 since x must be positive).

Is there a region of overlap in these two ranges i.e. can both inequalities hold simultaneously for some values of x? Yes, they can hold for 0 < x < 1/?2. Hence, x^2 < 2x < 1/x will be true for the range 0 < x < 1/?2. So this could be the correct ordering. Let’s go on to the next complex inequality.

(II) x^2 < 1/x < 2x

Again, let’s break up the inequality into two parts:

(i) x^2 < 1/x

x^1 < 1/x is rewritten as x^3 – 1 < 0 which gives us x < 1.

(ii) 1/x < 2x

1/x < 2x is rewritten as x^2 – 1/2 > 0 which gives us x < -1/?2 (not possible since x must be positive) or x > 1/?2

Can both x < 1 and x > 1/?2 hold simultaneously? Sure! For 1/?2 < x < 1, both inequalities will hold and hence x^2 < 1/x < 2x will be true. So this could be the correct ordering too.

(III) 2x < x^2 < 1/x

The inequalities here are:

(i) 2x < x^2

2x < x^2 can be rewritten as x(x – 2) > 0 which gives us x < 0 (not possible) or x > 2.

(ii) x^2 < 1/x

x^2 < 1/x gives us x^3 – 1 < 0 i.e. x < 1

Can x be less than 1 and greater than 2 simultaneously? No. Therefore, 2x < x^2 < 1/x cannot be the correct ordering.

Answer (D)

Is this method simpler?



does anyone understand why this method works? and also bunel evaluated each a<b<c as a<b and b<c why not evaluate a<c?
Intern
Intern
Joined: 11 Oct 2020
Posts: 6
Own Kudos [?]: 5 [0]
Given Kudos: 4
Location: United States
Concentration: General Management, Finance
WE:General Management (Insurance)
Send PM
Re: Inequalities Made Easy! [#permalink]
Hello,

Thanks for the amazing analysis on the inequalities. I have a little question.
The questions discussed are generally inequalities with only 1 variable x and its different functions.
What is the standard approach for the questions including different variables and their functions.
Is plugging values the only solution there?

Posted from my mobile device
Tutor
Joined: 17 Jul 2019
Posts: 1304
Own Kudos [?]: 2287 [0]
Given Kudos: 66
Location: Canada
GMAT 1: 780 Q51 V45
GMAT 2: 780 Q50 V47
GMAT 3: 770 Q50 V45
Send PM
Re: Inequalities Made Easy! [#permalink]
Expert Reply
Plugging values is almost never necessary. Check out 21:03 here:
https://youtu.be/Ob4KOxMtHf4

Posted from my mobile device
Intern
Intern
Joined: 11 Oct 2020
Posts: 6
Own Kudos [?]: 5 [0]
Given Kudos: 4
Location: United States
Concentration: General Management, Finance
WE:General Management (Insurance)
Send PM
Inequalities Made Easy! [#permalink]
@avigutmann,
Hello sir

I did refer to the video. I believe I should be specific regarding the question so that you can understand my problem statement better.

For example: If a < b < c , which of the following must be true?

(I) a < b^2
(II) b − a < c
(III) a^2 < b^2 < c^2

A. None
B. I only
C. II only
D. III only
E. II and III

----
Different variables and different functions. What should be the standard approach here (other than plugging values)?
Tutor
Joined: 17 Jul 2019
Posts: 1304
Own Kudos [?]: 2287 [0]
Given Kudos: 66
Location: Canada
GMAT 1: 780 Q51 V45
GMAT 2: 780 Q50 V47
GMAT 3: 770 Q50 V45
Send PM
Re: Inequalities Made Easy! [#permalink]
Expert Reply
You just have to draw them on a number line and do some thinking...
(I) we know a<b. What happens to b when it is squared? Will it move to the left? To the right? Stay in the same spot?
Depends on its position relative to -1, 0, and 1.
(II) b-a represents the (positive) distance between a and b. How does the value of c compare to the distance between a and b? Do we know anything about that distance? No...
(III) Okay so if we take three random numbers on the number line, would squaring them keep their original order, or would it potentially mess up the order? Again, depends on where they sit relative to -1, 0, and 1, because squaring a number will have very different effects in those different ranges.
Please let me know your thoughts fitshot.
Intern
Intern
Joined: 11 Oct 2020
Posts: 6
Own Kudos [?]: 5 [0]
Given Kudos: 4
Location: United States
Concentration: General Management, Finance
WE:General Management (Insurance)
Send PM
Inequalities Made Easy! [#permalink]
AviGutman
That's just awesome! You just widened my approach to these questions. Surely gonna help. Appreciate it!
However, I am feeling that this approach might have got some limited applicability or will consume more time on some complex questions (involving complex functions). I would love to be proved wrong :)

Can you help me with the same approach for this ques please:
If x,y,z>0 and x>y>z, which of the following could be true:
1: sqroot(x)>sqroot(y)>sqroot(z)
2: sqroot(z)>sqroot(y)>sqroot(x)
3: sqroot(x)>z^1/3> sqroot(y)
Senior Manager
Senior Manager
Joined: 09 Oct 2017
Posts: 250
Own Kudos [?]: 122 [0]
Given Kudos: 64
Location: Pakistan
Concentration: Finance, Strategy
GMAT 1: 640 Q48 V31
GRE 1: Q169 V160
GPA: 2.83
Send PM
Re: Inequalities Made Easy! [#permalink]
Bunuel wrote:
Some Inequalities, Mods and Sets

BY KARISHMA, VERITAS PREP


Here, let’s look at a question that involves inequalities and modulus and is best understood using the concept of sets. It is not a difficult question but it is still very tricky. You could easily get it right the first time around but if you get it wrong, it could take someone many trials before he/she is able to convince you of the right answer. Even after I write a whole post on it, I wouldn’t be surprised if I see “but I still don’t get it” in the comments below!

Anyway, enough of introduction! Let’s get to the question now.

Question: If \(\frac{x}{|x|} < x\), which of the following must be true about x?
(A) x > 1
(B) x > -1
(C) |x|< 1
(D) |x| = 1
(E) |x|^2 > 1

Solution:
First thing we do is tackle the mod. We know that |x| is just the absolute value of x.

So, \(\frac{x}{|x|}\) can take only 2 values: 1 or -1
If x is positive, \(\frac{x}{|x|} = 1\) e.g. if \(x = 4\), then \(\frac{4}{|4|} = 1\)
If x is negative, \(\frac{x}{|x|} = -1\) e.g. if \(x = -4\), then \(\frac{-4}{|-4|} = \frac{-4}{4} = -1\)
x cannot be 0 because we cannot have 0 in the denominator of an expression.

Now let’s work on the inequality.

\(x >\frac{x}{|x|}\) implies \(x > 1\) if x is positive or \(x > -1\) if x is negative.

Hence, for this inequality to hold, either \(x > 1\) (when x is positive) or \(-1 < x< 0\) (when x is negative)

x can take many values e.g. -1/3, -4/5, 2, 5, 10 etc.

Now think – which of the following MUST BE TRUE about every value that x can take?
(A) x > 1
or
(B) x > -1

I hope that you agree that x > 1 doesn’t hold for every possible value of x whereas x > -1 holds for every possible value of x. Mind you, every value greater than -1 need not be a possible value of x.

This concept might need some more work. Let me explain with another example.

Forget this question for a minute. Say instead you have this question:

Example 1: x > 2 and x < 7. What integral values can x take?
I guess most of you will come up with 3, 4, 5, 6. That’s correct. I can represent this on the number line.



The top arrow shows x > 2 and the bottom arrow shows x < 7. You see that the overlapping area includes 3, 4, 5 and 6. That is the region that satisfies both the inequalities.

Now consider this:

Example 2: x > 2 or x > 5. What integral values can x take?
Let’s draw that number line again.



So is the solution again the overlapping numbers i.e. all integers greater than 5? No. This question is different. x is greater than 2 OR greater than 5. This means that if x satisfies at least one of these conditions, it is included in your answer. Think of sets. AND means it should be in both the sets (i.e. the overlapping part) as was the case in example 1. OR means it should be in at least one of the sets. Hence, which values can x take? All integral values starting from 3 onwards i.e. 3, 4, 5, 6, 7, 8, 9 …

Now go back to this question. The solution is a one liner.
\(\frac{x}{|x|}\) is either 1 or -1.
So \(x > 1\) or \(x > -1\)
So which values can x take? All values included in at least one of the sets. Therefore, x > -1.

Note that the confusion lies only between the first two options. The other three options are rejected outright.
(C) |x|< 1 implies -1 < x < 1. Definitely doesn’t hold.
(D) |x| = 1 implies x = 1 or -1. Definitely doesn’t hold.
(E) |x|^2 > 1 implies either x < -1 or x > 1. Definitely doesn’t hold.

So what do you say? Are you convinced that the answer is (B)? This question is discussed HERE.

Attachment:
Ques51.jpg
Attachment:
Ques4 (1).jpg


Can we do this by squaring both sides and through a wavy line method?
Manager
Manager
Joined: 12 May 2021
Posts: 74
Own Kudos [?]: 6 [0]
Given Kudos: 126
Send PM
Inequalities Made Easy! [#permalink]
for equation x>x/|x| Can anyone any algebraically prove that if x is positive x>1 and if x is negative x>-1;
Intern
Intern
Joined: 18 Jul 2013
Posts: 6
Own Kudos [?]: [0]
Given Kudos: 6
Location: India
Send PM
Re: Inequalities Made Easy! [#permalink]
Hi Where can i find more resources on modulus, word problems, number properties (odd/even, factors, LCM etc.) similar to the one above shared on Inequalities? Bunuel

Thank you
Math Expert
Joined: 02 Sep 2009
Posts: 92915
Own Kudos [?]: 618955 [0]
Given Kudos: 81595
Send PM
Re: Inequalities Made Easy! [#permalink]
Expert Reply
AK008 wrote:
Hi Where can i find more resources on modulus, word problems, number properties (odd/even, factors, LCM etc.) similar to the one above shared on Inequalities? Bunuel

Thank you


Check the links below:
ALL YOU NEED FOR QUANT ! ! !
Ultimate GMAT Quantitative Megathread

Hope it helps.
User avatar
Intern
Intern
Joined: 03 Aug 2023
Posts: 1
Own Kudos [?]: 0 [0]
Given Kudos: 0
Send PM
Re: Inequalities Made Easy! [#permalink]
Is it fair to say that while a>b and c>d for non-negative numbers gives ac>bd, the reverse isn't necessarily true.
In other words, ac>bd does not imply that a>b and c>d or a>d and c>b?

For example, a=6,b=2,c=3,d=8:
ac = 18, bd = 16. Therefore, ac > bd

But, c<d and a<d, so therefore the "split inequalities" do not hold.

I find this strange so just want to check whether I have missed something here?
GMAT Club Bot
Re: Inequalities Made Easy! [#permalink]
   1   2   3 
Moderator:
Math Expert
92914 posts

Powered by phpBB © phpBB Group | Emoji artwork provided by EmojiOne